3
$\begingroup$

For $s \in (0,1)$, we consider the spectral fractional Laplacian \begin{align} (-\Delta)^{-s}u = \sum_{k=1}^{\infty}\lambda_k^{-s}(\phi_k,u)_{L^2}\phi_k \end{align} where \begin{align*} \begin{cases} -\Delta\phi_k = \lambda_k\phi_k &\mbox{in }\Omega,\\ \partial_\nu \phi_k = 0 &\mbox{on }\partial\Omega,\qquad k\geq 1. \end{cases} \end{align*} Then, for $\alpha \in (0,1)$, we define the space \begin{align*} H^\alpha(\Omega) = \{ u\in L^2(\Omega):~ \nabla (-\Delta)^{(\alpha-1)/2}u\in L^2(\Omega) \}, \end{align*} and the space \begin{align*} \tilde H^\alpha(\Omega) = \{ u\in L^2(\Omega)~:~ \sum_{j=1}^\infty \lambda_j^\alpha|(\phi_j,u)_{L^2(\Omega)}|^2<\infty\} \end{align*}

Questions:

  1. Are these two spaces equivalent?

  2. Do Sobolev embeddings, Poincaré inequalities, etc hold in them? If yes, where can I find a reference?


UPDATE. After the discussion in the comments, let me reformulate the question in a way that avoids (hopefully) some issues:

Let $(\phi_k)\subset L^2(\Omega)$ be a complete orthonormal system for $\mathcal L^2(\Omega)= \{ u\in L^2(\Omega):\int_\Omega u d x =0 \}$ composed of eigenfunctions of $-\Delta$ with homogeneous Neumann boundary conditions with eigenvalues $(\lambda_k)\subset (0,\infty)$.

Q1: Is \begin{align} (-\Delta)^{-s}u = \sum_{k=1}^{\infty}\lambda_k^{-s}(\phi_k,u)_{L^2}\phi_k \end{align} well-defined?

Q2: Define \begin{align*} \mathcal H^\alpha(\Omega) = \{ u\in \mathcal L^2(\Omega):~ \nabla (-\Delta)^{(\alpha-1)/2}u\in L^2(\Omega) \}, \end{align*} and the space \begin{align*} \tilde{\mathcal H}^\alpha(\Omega) = \{ u\in \mathcal L^2(\Omega)~:~ \sum_{j=1}^\infty \lambda_j^\alpha|(\phi_j,u)_{L^2(\Omega)}|^2<\infty\} \end{align*} Are these spaces well-defined and are they equivalent?

$\endgroup$
17
  • 1
    $\begingroup$ Not sure what $(-\Delta)^{-s}$ does to the constant function $\phi_1$ (which corresponds to $\lambda_1 = 0$). If $\phi_1$ is not in the domain of $(-\Delta)^{-s}$, then it belongs to $\tilde{H}^\alpha$, but not to $H^\alpha$, I guess. $\endgroup$ Sep 16, 2022 at 14:06
  • $\begingroup$ @MateuszKwaśnicki I'm not quite sure what you mean. What is the domain of $(-\Delta)^{-s}$? Why are constants in $\tilde H^\alpha$ but not in $H^\alpha$? $\endgroup$
    – Zac
    Sep 16, 2022 at 14:42
  • 1
    $\begingroup$ Just asking for clarification. Currently, as far as I understand, we have $\lambda_1 = 0$ and the term $k = 1$ in the definition of $(-\Delta)^{-s}$ involves $\lambda_1^{-s} = 0^{-s}$. $\endgroup$ Sep 16, 2022 at 19:32
  • $\begingroup$ @MateuszKwaśnicki is right: the Neumann laplacian is not invertible because its leading eigenvalue is zero. $\endgroup$ Sep 17, 2022 at 10:49
  • $\begingroup$ Here are two possibilities for correcting this bug with the leading eigenvalue to avoid division by zero: (1) you can change the boundary conditions to Dirichlet or (2) you can shift the operator as in, e.g., Section 3.2 of link.springer.com/article/10.1007/s40072-020-00175-6 $\endgroup$ Sep 17, 2022 at 14:22

1 Answer 1

4
+25
$\begingroup$

Yes, they are. Since $(\phi_j,(-\Delta)^\eta u) = \lambda_j^\eta (\phi_j,u)$, it suffices to consider the case $\alpha = 1$. Since, for any closed operator $A$, the domain of $A$ coincides with that of $(A^*A)^{1/2}$ (their graph norms are the same), it remains to note that, for $\nabla$ as in the question, $\nabla^*\nabla$ coincides with the Neumann Laplacian, which is the case if $\Omega$ is regular enough.

$\endgroup$
2
  • $\begingroup$ Hi Martin, How are you? It seems $\nabla$ is a derivative operator and not endowed with any boundary conditions. So, it's unclear why "$\nabla^*\nabla$ coincides with the Neumann Laplacian." That's why the answer I provided discussed the boundary terms that arise from integrating by parts. Not sure why those boundary terms are necessarily bounded for functions in $H^{\alpha}(\Omega)$ ... $\endgroup$ Sep 16, 2022 at 10:18
  • 1
    $\begingroup$ @NawafBou-Rabee Take the case of $\Omega = [0,1]$. By definition, the domain of $\nabla^*$ consists of those $f \in L^2$ such that there exists $g \in L^2$ with $(g,u) = (f,\nabla u)$ for all $u$ in the domain of $\nabla$. For $f$ smooth, this forces it to vanish at the boundary since otherwise a boundary term appears which cannot be generated by $g$. As a consequence, functions in the domain of $\nabla^*\nabla$ must have vanishing gradient at the boundary. $\endgroup$ Sep 16, 2022 at 13:21

Your Answer

By clicking “Post Your Answer”, you agree to our terms of service and acknowledge you have read our privacy policy.

Not the answer you're looking for? Browse other questions tagged or ask your own question.